Jump to content

Lorenz Force thrust


DandelionTheory

Recommended Posts

 

Lines AB and BC are wires connected at point B and have an air gap between points A and C of 1inch with a potential difference of 30kv.

1598801880_20200205_0310512.thumb.jpg.80d62c9efb96d835d5be0cc8dd4ad34e.jpg

While electrons arc between points A and C they experience the lorentz force and have a curved path due to the magnetic fields of the current in the wires.

So the curved path with radius r needs a few things: 

The curve of electrons from point A to point C can be found by solving for r, which is dependent on the magnitude of the magnetic field from the current in the wires AB and BC. An arc's current increases exponentially with time and therefore the current in the system will increase while an arc is in place, and therefore r will increase.

I do not know off hand how to calculate over time, I am aware r will change as current increases.

I'm having trouble solving drift velocity of the electrons arcing between points A and C not in a wire, if anyone can point me to the right place to look I would appreciate it.

Edited by DandelionTheory
Link to comment
Share on other sites

43 minutes ago, DandelionTheory said:

 

Lines AB and BC are wires connected at point B and have an air gap between points A and C of 1inch with a potential difference of 30kv.

1598801880_20200205_0310512.thumb.jpg.80d62c9efb96d835d5be0cc8dd4ad34e.jpg

While electrons arc between points A and C they experience the lorentz force and have a curved path due to the magnetic fields of the wires.

So the curved path with radius r needs a few things: 

The curve of electrons from point A to point C can be found by solving for r, which is dependent on the magnitude of the magnetic field from the current in the wires AB and BC. An arc's current increases exponentially with time and therefore the current in the system will increase while an arc is in place, and therefore r will increase.

I do not know off hand how to calculate over time, I am aware r will change as current increases.

I'm having trouble solving drift velocity of the electrons arcing between points A and C not in a wire, if anyone can point me to the right place to look I would appreciate it.

So this is the drawing to analyze - no changing it.

There are electrons in an arc - meaning they have left the wire, and the wire is positively charged. The electrons will feel an electrostatic attraction to the wire, and the wire therefore feels an force toward the electrons.

Where have you accounted for that force?

 

 

 

Link to comment
Share on other sites

28 minutes ago, swansont said:

So this is the drawing to analyze - no changing it.

There are electrons in an arc - meaning they have left the wire, and the wire is positively charged. The electrons will feel an electrostatic attraction to the wire, and the wire therefore feels an force toward the electrons.

Where have you accounted for that force?

 

 

 

Youre assuming I mean all the electrons in a wire leave and this is some magic thing. No.

Assume point A is battery negative and point C is battery positive. Also assume more than enough energy is in the systems battery for thousands of "cycles"( I didn't calculate for time yet)

Do you assume this force exceeds the force on each wire?

Edited by DandelionTheory
Link to comment
Share on other sites

35 minutes ago, DandelionTheory said:

Youre assuming I mean all the electrons in a wire leave and this is some magic thing. No.

Assume point A is battery negative and point C is battery positive. Also assume more than enough energy is in the systems battery for thousands of "cycles"( I didn't calculate for time yet)

Charge is conserved. If electrons leave the wire, the wire must be positively charged. Even if there's a battery in there, the excess charge will not magically be constrained to the battery.

 

35 minutes ago, DandelionTheory said:

Do you assume this force exceeds the force on each wire?

I made no assumptions about that. I am asking you for your analysis of this, as part of your conjecture. I am pointing out that this analysis is missing. You own the burden of proof here.

Link to comment
Share on other sites

11 minutes ago, swansont said:

Charge is conserved. If electrons leave the wire, the wire must be positively charged

Does this mean arc's cannot form naturally or the opposite force of an electron leaving a wire attracts it back? The electric field is constant in the example until the end of the cycle. Between points A and C (Which is not shown)

11 minutes ago, swansont said:

I am pointing out that this analysis is missing

Please be specific.

 

Edited by DandelionTheory
Link to comment
Share on other sites

1 hour ago, DandelionTheory said:

Does this mean arc's cannot form naturally or the opposite force of an electron leaving a wire attracts it back?

If an electron leaves the wire there is a +1e charge in the wire. You have a great many electrons leaving the wire — a current I, flowing at some speed over some distance. They flow to the other side because of the potential difference. There will be an electrostatic attraction between the charges and the wire.

 

1 hour ago, DandelionTheory said:

The electric field is constant in the example until the end of the cycle. Between points A and C (Which is not shown)

This is (supposed to be) physics. You have to justify that this will be the case.

 

1 hour ago, DandelionTheory said:

Please be specific.

There is nothing on your page about the forces the electrons exert on the wires. There's a qE term in your other forces, but there is no analysis of the arc. You can't just ignore it. To do so leaves you with the silly conclusion that there is a net force that would allow propulsion, in violation of the laws of motion.

Link to comment
Share on other sites

 

On 1/31/2020 at 2:14 PM, DandelionTheory said:

Mass is ejected in the process

As far as I can tell mass is ejected and then returned to the system. Your drawing shows ejection of mass (electrons) at one point (A) and then the electrons are recovered at another point (B). Electrons seem to be circulating in the system.

 

Link to comment
Share on other sites

1 hour ago, swansont said:

If an electron leaves the wire there is a +1e charge in the wire. You have a great many electrons leaving the wire — a current I, flowing at some speed over some distance. They flow to the other side because of the potential difference. There will be an electrostatic attraction between the charges and the wire.

Thank you, can you show me an example of that force?

1 hour ago, swansont said:

You have to justify that this will be the case

Thank you.

1 hour ago, swansont said:

There is nothing on your page about the forces the electrons exert on the wires. There's a qE term in your other forces, but there is no analysis of the arc. 

Force on the arc from the wires and vice versa?

Agreed. I thought it had to do with current phase (like a cavity magnetron but this is represented as a DC "wiggle" ) and assumed it required integration to calculate potential path of the electron while in the arc. Which I mentioned needing help with. (Again I assumed it was part of the answer)

If a wire is placed between points A and C, I can use it as a maximum value to derive the affect of a straight arc on the wires. I am still working out a minimum approximation.

55 minutes ago, Ghideon said:

As far as I can tell mass is ejected and then returned to the system.

Hrm, mass ejecting mass is a reaction, while mass colliding with mass is another reaction. I'm assuming this satisfies your inquiry.

Link to comment
Share on other sites

34 minutes ago, DandelionTheory said:

Hrm, mass ejecting mass is a reaction, while mass colliding with mass is another reaction. I'm assuming this satisfies your inquiry.

Depends on what you wish to achieve. Propulsion of the system is not possible if the mass is recycled in the system.

Link to comment
Share on other sites

18 minutes ago, swansont said:

It's the coulomb force.

F = kq1q2/r^2

Thank you I'll work on it.

10 minutes ago, Ghideon said:

Depends on what you wish to achieve. Propulsion of the system is not possible if the mass is recycled in the system.

I wonder if you're considering the angle of interaction. It's set up to eject electrons downward-ish and receive them as they come up. The magnetic fields from the wires are meant to curl the arc so the mass to mass interactions are angled vertically.

Link to comment
Share on other sites

43 minutes ago, DandelionTheory said:

I wonder if you're considering the angle of interaction. It's set up to eject electrons downward-ish and receive them as they come up. The magnetic fields from the wires are meant to curl the arc so the mass to mass interactions are angled vertically.

Angle does not matter. For any mass that is ejected and then returened the effects from ejection and collection will cancel. 

Link to comment
Share on other sites

3 hours ago, Ghideon said:

For any mass that is ejected and then returened the effects from ejection and collection will cancel. 

Clarification: by effects I specifically mean effects on momentum of the system. 

In reality, and more generally, losses due to heat or resistance or other from emitting electron at point A are not cancelled by receiving electrons at point C. 

Link to comment
Share on other sites

9 hours ago, Ghideon said:

 

In reality, and more generally, losses due to heat or resistance or other from emitting electron at point A are not cancelled by receiving electrons at point C. 

I'll be more specific.

Due to mass not leaving the system, just the wires (it comes back), I never added it to the net force on the wires. I admit it's not thorough. I did use it as an explanation to how it works because it's not attached to a wire, it ejects the charged mass outside the wires to lose the drag coefficient of the Laplace force on the wires. 

My idea came fromI a rail gun: the rails, slug, and back end make a current loop that induce the Laplace force on each current. The force is equal and opposite in each direction away from the center of the current loop. So if the slug is an arc of current, any charge carriers accelerated too far to return to the system would be lost mass and energy. I'm attempting to show the resultant Laplace force on the wires due to the current loop seems to favor one side. But the issue I'm having (I assume) involves calculating the possible position of the electron during the "cycle" so I can better clarify the magnitude of the Laplace force on the wires from the arc.

Link to comment
Share on other sites

Not sure I understand what you try to show. Are you trying to show that the circulating electrons mass generates thrust?

 

1 hour ago, DandelionTheory said:

any charge carriers accelerated too far to return to the system would be lost mass and energy.

From what I see so far this is the only thing in the setup that could create (a tiny) thrust. Calculating the momentum of the lost mass gives the momentum of the rest of the system in the opposite direction.

 

 

Link to comment
Share on other sites

51 minutes ago, Ghideon said:

Are you trying to show that the circulating electrons mass generates thrust?

No. I'm trying to show the the lorentz force pushes all currents from the center of the loop, the Laplace force calculates the physical translation of the wires. If the arc were in a field that accelerated it far beyond the point of return to the cycle, that mass ejection would add to the force on the wires in the opposite direction. Mass ejection is a symptom of operation at high particle velocity.

Link to comment
Share on other sites

4 hours ago, DandelionTheory said:

I'll be more specific.

Due to mass not leaving the system, just the wires (it comes back), I never added it to the net force on the wires. I admit it's not thorough. I did use it as an explanation to how it works because it's not attached to a wire, it ejects the charged mass outside the wires to lose the drag coefficient of the Laplace force on the wires. 

My idea came fromI a rail gun: the rails, slug, and back end make a current loop that induce the Laplace force on each current. The force is equal and opposite in each direction away from the center of the current loop. So if the slug is an arc of current, any charge carriers accelerated too far to return to the system would be lost mass and energy. I'm attempting to show the resultant Laplace force on the wires due to the current loop seems to favor one side. But the issue I'm having (I assume) involves calculating the possible position of the electron during the "cycle" so I can better clarify the magnitude of the Laplace force on the wires from the arc.

If you want to (potentially) show thrust you have to account for all the forces, and by focusing on the Laplace force you aren't doing that.

Link to comment
Share on other sites

2 hours ago, swansont said:

If you want to (potentially) show thrust you have to account for all the forces, and by focusing on the Laplace force you aren't doing that.

Agree. And I think we know the result already, internal forces will cancel and generate zero thrust.

4 hours ago, DandelionTheory said:

I'm trying to show the the lorentz force pushes all currents from the center of the loop, the Laplace force calculates the physical translation of the wires.

You mean that the triangle shape will accelerate upwards when electrons emitted at A and recovered at C? 

image.png.488524f4b55a423294e5e0410a50ca12.png

 

Link to comment
Share on other sites

Create an account or sign in to comment

You need to be a member in order to leave a comment

Create an account

Sign up for a new account in our community. It's easy!

Register a new account

Sign in

Already have an account? Sign in here.

Sign In Now
×
×
  • Create New...

Important Information

We have placed cookies on your device to help make this website better. You can adjust your cookie settings, otherwise we'll assume you're okay to continue.